LSAT and Law School Admissions Forum

Get expert LSAT preparation and law school admissions advice from PowerScore Test Preparation.

 jharirchi
  • Posts: 1
  • Joined: Aug 24, 2012
|
#4967
It seems that none of the answers MUST BE TRUE on #2 on page 3-31. I understand the diagramming of the question, but it does not specify what is the correct answer and I cannot seem to find one that satisfies the question.
User avatar
 Dave Killoran
PowerScore Staff
  • PowerScore Staff
  • Posts: 5852
  • Joined: Mar 25, 2011
|
#4970
Hey Jharirchi,

Take a look at page 3-107, in the middle of the page. The answer and explanation for that question is given there. The correct answer is (C), for this reason:

When C is shown third, A must be shown second, and D must be shown fourth. Because D and E cannot be shown consecutively, E cannot be shown fifth, and must be shown first. Thus, answer choice (C) must be true and is correct.

Please let me know if that helps. Thanks!

Get the most out of your LSAT Prep Plus subscription.

Analyze and track your performance with our Testing and Analytics Package.